A farmer wants to build a rectangular fence to enclose a 144\ \textup{ft}^2 yard. If the yard is square, how many feet of fence will he need?

Answers

Answer 1

Answer:

12* 12 = 144

thus 4*12 = 48 feet of fence

Step-by-step explanation:


Related Questions

Suppose that the functions and g are defined for all real numbers x as follows.
f(x)=x+6
g(x) = 2x + 6
Write the expressions for (f-g)(x) and (fg)(x) and evaluate (f+g)(1).

Answers

Answer:

Step-by-step explanation:

Given functions are,

f(x) = x + 6

g(x) = 2x + 6

(f - g)(x) = (x + 6) - (2x + 6)

                   = -x

(f . g)(x) = f(x) × g(x)

            = (x + 6)(2x + 6)

            = 2x² + 6x + 12x + 36

            = 2x² + 18x + 36

(f + g)(x) = (x + 6) + (2x + 6)

             = 3x + 12

(f + g)(1) = 3(1) + 12

            = 15  

Find the equation of the line passing through (4,1) and perpendicular to the line whose equation is 1x-3y-4=0

Answers

9514 1404 393

Answer:

  3x +y -13 = 0

Step-by-step explanation:

The perpendicular line will have the variable coefficients swapped and one of them negated. The new constant will be appropriate to the given point.

  3(x -4) +1(y -1) = -0

  3x +y -13 = 0

_____

Additional comment

The given equation is in "general form", so that is the form of the equation we have given as the answer. This form is convenient in that the general form equation for a line through the origin, ax+by=0, is easily translated to make it pass through a point (h, k): a(x -h) +b(y -k) = 0. Eliminating parentheses puts the equation back into general form.

what's the difference between -1/2 and 1/6

Answers

Answer: -4/6 or -2/3

Step-by-step explanation:

First, you find a common denominator among the fractions, which would be 6.

Convert -1/2 to have 6 as its denominator.

-1/2* 3/3 = -3/6

And then subtract them.

-3/6 - 1/6 = -4/6

-4/6 simplified is -2/3

When you subtract a positive number from a negative number, you are adding their absolute values.

What is the slope of the line?
-3

-1/3

1/3

3

Answers

Answer:

D) 3

Step-by-step explanation:

Rise/run, rise is 3, run is 1

Answer:

3

Step-by-step explanation:

Pick two points on the line

(0,0) and ( 1,3)

The slope is found by

m = ( y2-y1)/(x2-x1)

    = ( 3-0)/(1-0)

    = 3/1

   = 3

Y
X

Pls help me you’ll get 29 points

Answers

Answer:

x = 60

Step-by-step explanation:

The sum of the angles of a triangle add to 180

x+x+x = 180

3x = 180

Divide by 3

3x/3 =180/3

x = 60

What is the value of the expression 10(n-6) when 4=14

Answers

Answer:

it's going to be 4n so its going to be 4...

Step-by-step explanation:

Answer:

80

Step-by-step explanation:

Assuming you meant when n=14

10(n-6)

plug in 14 for n

10(14-6)

work out parenthesis first

10(8)=

80

Use the​ Gauss-Jordan method to solve the system of equations. If the system has infinitely many​ solutions, give the solution with z arbitrary

Answers

Write each equation in standard form:

3x + y + 3z = 11

x + 2y + z = 7

-x + y + z = 0

In matrix form, this is

[tex]\begin{bmatrix}3&1&3\\1&2&1\\-1&1&1\end{bmatrix}\begin{bmatrix}x\\y\\z\end{bmatrix}=\begin{bmatrix}11\\7\\0\end{bmatrix}[/tex]

and in augmented matrix form,

[tex]\left[\begin{array}{ccc|c}3&1&3&11\\1&2&1&7\\-1&1&1&0\end{bmatrix}\right][/tex]

Now for the row operations:

• Add row 1 to -3 (row 2), and add row 1 to 3 (row 3):

[tex]\left[\begin{array}{ccc|c}3&1&3&11\\0&-5&0&-10\\0&4&6&11\end{bmatrix}\right][/tex]

• Multiply row 2 by -1/5:

[tex]\left[\begin{array}{ccc|c}3&1&3&11\\0&1&0&2\\0&4&6&11\end{bmatrix}\right][/tex]

• Add -4 (row 2) to row 3:

[tex]\left[\begin{array}{ccc|c}3&1&3&11\\0&1&0&2\\0&0&6&3\end{bmatrix}\right][/tex]

• Multiply row 3 by 1/6:

[tex]\left[\begin{array}{ccc|c}3&1&3&11\\0&1&0&2\\0&0&1&\frac12\end{bmatrix}\right][/tex]

• Add -1 (row 2) and -3 (row 3) to row 1:

[tex]\left[\begin{array}{ccc|c}3&0&0&\frac{15}2\\0&1&0&2\\0&0&1&\frac12\end{bmatrix}\right][/tex]

• Mutiply row 1 by 1/3:

[tex]\left[\begin{array}{ccc|c}1&0&0&\frac52\\0&1&0&2\\0&0&1&\frac12\end{bmatrix}\right][/tex]

Then the solution to the system is (x, y, z) = (5/2, 2, 1/2).

Find the product of these complex numbers.
(8 + 5)(6 + 3) =

Answers

The Answer is D, 33+54i!
Answer:

D

Step-by-step explanation:

For this kind of stuff, we’ll need to use the FOIL Method. Basically start with the Firsts, Outers, Inners and Lasts.

(8 + 5i) (6 + 3i) =

Multiply 8 x 6

Answer: 48

Multiply 5i x 3i

Answer: 15i^2

Multiply 5i x 6

Answer: 30i

Multiply 8 x 3i

Answer: 24i

Then add 30i + 24i = 54i
Finally subtract 48 - 15 = 33

Final Answer: 33 + 54i

Classify the type of angle.
A: Acute
B: Right
C: Straight
D: Obtuse

Answers

Answer:

Obtuse

Step-by-step explanation:

I really don’t know which angle they’re asking for because the 140 degree angle is obtuse and the 40 degree angle is acute

Answer:

obtuse angle that's the answer

821) The integon which is 15 more than - 55 is

Answers

Answer:

-40

Step-by-step explanation:

-55 + 15 = x

-40 =x

Solve the inequality (help please)

Answers

Answer:

v<1 23/25

Step-by-step explanation:

The inequality simplifies to 48/25, which is equivalent to 1 23/25.

Write the equation of the line in point-slope form given the information:

Slope = -1/5

Y intercept = -3

Answers

point slope form:

y + 3= -1/5 x (x+0)

Step-by-step explanation:

-use the point slope form equation: y - y1 = m (x - x1).

-using the given information we know that m = -1/5 and that the y intercept is (0,-3).

- y and x will be kept as a variable.

-plug in the y1 and x1 from the y intercept:

y + 3 = m(x + 0)

-once you've plugged in the y intercept then plug in the slope which gives you the answer:

y + 3= -1/5 x (x+0)

I need help:/ I’m in college

Answers

Step-by-step explanation:

Amount of acid = 14.9% of 331 mL solution

= 0.149×(331 mL)

= 49.3 mL acid

Blood pressure values are often reported to the nearest 5 mmhg (100, 105, 110, etc.). the actual blood pressure values for nine randomly selected individuals are given below.

108.6 117.4 128.4 120.0 103.7 112.0 98.3 121.5 123.2

Required:
a. What is the median of the reported blood pressure values?
b. Suppose the blood pressure of the second individual is 117.7 rather than 117.4 (a small change in a single value). What is the new median of the reported values?
c. What does this say about the sensitivity of the median to rounding or grouping in the data?

Answers

Answer:

Step-by-step explanation:

Arranging the data in the ascending order:

108.6 98.3 103.7 112 117.4 120 121.5 123.2 128.4

The median is the middle value of the data set:

a)

Hence,

median = 117.4

b)

When the value of blood pressure is 117.7 instead of 117.4 then the median will be:

Median = 117.7

c)

This indicates that the median of a well sorted set of data is depends upon the middle value of the data set.

Answer question below

Answers

Answer:

edge of cube = root 13.5/6 = 1.5

volume of cube = (Edge of cube)^3= (1.5)^3 = 3.37500 m^3

PLEASE PLEASE HELP ASAPPPP IM BEING TIMEDDD

6x2y − 3xy − 24xy2 + 12y2
Rewrite the expression completely factored. Show the steps of your work.

Answers

Answer:

3y(2x-1)(x-4y)

Step-by-step explanation:

Apply exponent rule:

6x^2y-3xy-24xyy+12yy

Rewrite 12 as 4*3

Rewrite -24 as 8*3

Rewrite 6 as 2*3

2*3x^2y-3xy+8*3xyy+4*3yy

Factor out common term 3y:

3y(2x^2-x-8xy+4y)

Factor 2x^2-x-8xy+4y:

3y(2x-1)(x-4y)

Your Answer Is 3y(2x-1)(x-4y)

4. Five cards are randomly chosen from a deck of 52 (13 denominations with 4 suits). a. How many ways are there to receive 5 cards from a deck of 52

Answers

Answer:

There are 2,598,960 ways to receive 5 cards from a deck of 52.

Step-by-step explanation:

The order in which the cards are chosen is not important, which means that the combinations formula is used to solve this question.

Combinations formula:

[tex]C_{n,x}[/tex] is the number of different combinations of x objects from a set of n elements, given by the following formula.

[tex]C_{n,x} = \frac{n!}{x!(n-x)!}[/tex]

a. How many ways are there to receive 5 cards from a deck of 52?

[tex]C_{52,5} = \frac{52!}{5!(47)!} = 2598960[/tex]

There are 2,598,960 ways to receive 5 cards from a deck of 52.

The diameter of the base is the cone measured 8 units. The height measures 6 units.

What is the volume of the cone?

A) 24 π cubic units

B) 32 π cubic units

C)48 π cubic units

D)64 π cubic units

Answers

The answer is B) 32 cubic units

What is the range for the following set of numbers?57, -5, 11, 39, 56, 82, -2, 11, 64, 18, 37, 15, 68

Answers

The range= the highest number - the lowest number

so

82-(-2)

=84

then ur answer is 84

If 400 patrons visit the park in March and 550 patrons visit in April, the total number of patrons who
visited the park over the two months falls into all of the following categories except
O real numbers
O rational numbers
o irrational numbers

Answers

950 is a real number, and it is a rational number since it can be expressed as 950/1. Therefore, it falls into all of the categories expect irrational numbers.

Help me with this please!!

Answers

Answer:

[tex]y=\sqrt{x} -2[/tex]

Step-by-step explanation:

If you were to replace x with 0 in each of the equations, you would get

from top to bottom choices:

[tex]\sqrt{2}[/tex]

[tex]\sqrt{-2}[/tex]

2

-2

Since we can see that the line touches the y-axis on the number -2, we know that it is the last choice that is the answer

Two planes are the same altitude. From the airport , one plane is 50 km away in the direction of N°60 E and another is 80 km away in the direction of S50° E .How far apart are the two planes

Answers

9514 1404 393

Answer:

  78.5 km

Step-by-step explanation:

Measured at the airport, the angle between the two planes is ...

  180° -60° -50° = 70°

The law of cosines tells us the distance between the planes is ...

  d = √(50² +80² -2·50·80·cos(70°)) ≈ √6163.84 ≈ 78.5 . . . km

The planes are about 78.5 km apart.

Matt buys a new fish tank. The fish tank is in the shape of a cuboid. The diagram shows water in the tank. 30 cm 30 cm 100 cm Matt knows 1000 cm' = 1 litre 1 gallons = 4.5 litres He can keep 2 small fish in the tank for every 1 gallon of water in the tank. Matt thinks he can keep more than 36 small fish in the tank. Is Matt correct?​

Answers

Answer: Yes, but only if he houses 37, 38, 39, or 40 fish

Anything larger than 40 and he'll need more room.

==========================================================

Explanation:

The tank is 30 cm by 30 cm by 100 cm. The volume is 30*30*100 = 90,000 cm^3 which is shorthand for "cubic centimeters".

We're told that 1000 cm^3 = 1 liter, which means the 90,000 cm^3 converts to (90,000)/(1000) = 90 liters.

The fish tank is 90 liters.

Since 1 gallon = 4.5 liters, this means the 90 liter tank converts to 90/(4.5) = 20 gallons

----------------------------

Your teacher mentions "He can keep 2 small fish for every 1 gallon".

Since the tank is 20 gallons, that means he can keep 20*2 = 40 fish. This value is larger than 36, so Matt is correct to a point. If Matt is thinking 37, 38, 39, or 40 fish then he would be correct. If Matt is wanting more than 40 fish, then he'll need a bigger tank.

In short, he can't have any number over 36 and can only have 4 specific values (the four values mentioned earlier).

So technically, Matt is correct, but strong clarification is needed.

28, 45, 12, 34, 36, 45, 19,20
Part 1: Find the mean of this set of data.
Part 2: Find the mean absolute deviation of this set of data.
Part 3: Using complete sentences, explain what your results for part A and B mean.
i really need an answer for part c

Answers

Part A

To find the mean, we add up the values and divide by n = 8 since there are 8 values in this set.

Adding the values gets us

28+45+12+34+36+45+19+20 = 239

Dividing this over 8 then leads to 239/8 = 29.875

Answer: 29.875

============================================================

Part B

We'll subtract each data value from the mean. We apply absolute value to ensure the result is never negative.

|28 - 29.875| = 1.875 |45 - 29.875| = 15.125 |12 - 29.875| = 17.875 |34 - 29.875| = 4.125 |36 - 29.875| = 6.125 |45 - 29.875| = 15.125 |19 - 29.875| = 10.875 |20 - 29.875| = 9.875

The list of results we get so far is:

1.875, 15.125, 17.875, 4.125, 6.125, 15.125, 10.875, 9.875

This represents the distance each value is from the mean.

Add these values up and divide by n = 8

1.875+15.125+17.875+4.125+6.125+15.125+10.875+9.875 = 81

81/8 = 10.125

Answer: 10.125

============================================================

Part C

The result of part A is one way to measure the center of the distribution of values. It's the average value, which can more or less represent the entire group. Think of it being like how people vote in a senator to represent them in congress. Ideally, this senator is a supposed "average" person to represent everyone.

The result of part B builds on what part A found. The result of part B is the average distance each value is from the center. This is because each time we subtracted and applied absolute value, we found the distance that item was from the mean.

Example: The calculation |28 - 29.875| = 1.875 shows that 28 is exactly 1.875 units from the mean 29.875

By adding up those results and dividing by 8, we are finding the average distance from the mean. Effectively, it tells us how spread out the data set is. The mean absolute deviation (MAD) is a measure of spread in a similar fashion that the standard deviation is, or in a more looser sense, the range is as well.

---------------

In short, the result of part A is a measure of center while the result of part B is a measure of spread. I use "a" instead of "the" because there are other measures of center and other measures of spread.

Hello everyone can someone answer this question please

Answers

9514 1404 393

Answer:

  (a)  2

Step-by-step explanation:

Each inch is 2.54 cm, so 5.08 cm is ...

  x / (5.08 cm) = (1 in) / (2.54 cm)

  x = (1 in)(5.08/2.54) = (1 in)(2)

  x = 2 in

5.08 cm equals 2 inches.

Write the equation of each line in slope intercept form. Slope is -6, and (1,-2) is on the line

Answers

Y = -6x + 4

See the attached photo for further reference.

Hope this helps! Please make me the brainliest, it’s not necessary but appreciated, I put a lot of effort and research into my answers. Have a good day, stay safe and stay healthy.

what is the value of x, given that figure MNOP is a trapezoid with median qr A. 12 B. 8 C. 6 D. 16

Answers

The answer is D. Because if we put 16 in place of x, MP, NO will be 24, 16 respectedly

Step-by-step explanation:

So all horizontal line will be decrease from line MP to No.

As result MP will be = 24, QR = 20, NO = 16. We can see there's different of 4.

The value of x in the trapezoid is 16.

Option D is the correct answer.

What is a trapezium?

It is a quadrilateral that has one pair of parallel sides and a height.

The area is calculated as: 1/2 x sum of the parallel sides x height.

Examples:

Area of a trapezium that has the parallel sides as 3 cm and 4 cm and a heght o 5 cm.

Area = 1/2 x (3 + 4) x 5

Area = 1/2 x 7 x 5

Area = 35/2 = 17.5 cm^2

We have,

In a trapezoid, the median is the average of the parallel sides,

So we have:

QR

= (NO + MP)/2

= (x + x + 8)/2

= (2x + 8)/2

= x + 4

Since we also know that QR = 20, we can set the two expressions equal to each other and solve for x:

x + 4 = 20

x = 16

Therefore,

The value of x is 16.

Learn more about trapezium here:

https://brainly.com/question/22607187

#SPJ7

Find the lengths the missing side

Answers

Answer:

Long leg = 10√3Short leg = 10Hypotenuse = 20

Step-by-step explanation:

Concept:

Here, we need to know the idea of a special triangle 30-60-90.

A special right triangle is a right triangle with some regular feature that makes calculations on the triangle easier, or for which simple formulas exist.

The ratio between the corresponding side of each angle is 1 : √3 : 2

If you are still confused, please refer to the attachment below for a graphical explanation.

Solve:

30-60-90 ⇔ 1 : √3 : 2

Given the side corresponding to 90° is 20

30° : 90° = 1 : 2

30° : 20 = 1 : 2

30° = 10

30° : 60° = 1 : √3

10 : 60° = 1 : √3

60° = 10√3

Hope this helps!! :)

Please let me know if you have any questions

Yousef is making cookies for a friend's birthday. The recipe only makes one dozen cookies, and he wants to take 4 dozen to the birthday
party. If he needs 1 cups of flour to make one dozen, how much flour will he need to make 4 dozen? Simplify your answer and write it as
a mixed number

Answers

Answer:

6 and 2/3

Step-by-step explanation:

you do 1 2/3 x 4 because that is the amount of flour you need to make one batch. That is the correct answer.

PLEASE HELP ASAPPPPPP!!!! (answer in decimal)

Answers

Answer:

465/1178 = .395 = 39.5%

Step-by-step explanation:

        224 + 245

224+ 387 + 245 + 322

465/1178 = .395 = 39.5%

Other Questions
Find the missing measure if a and b are the legs of the right triangle and c is the hypotenuse, with a = 11 and c =18. Ella has two 8ft long boards she needs to cut pieces that are 15 inches long how many 15 inch pieces can she cut the two boards What is the difference between a hard drive and a tape drive? Hard drives use magnetic heads and are fast to write data; tape drives write to disks and are fast to retrieve data. Hard drives write to disks and are fast to retrieve data; tape drives use magnetic heads and are fast to write data. Hard drives use magnetic heads and are best for archival data; tape drives write to disks and are used for ongoing backups. Hard drives write to disks and are used for ongoing backups; tape drives use magnetic heads and are best for archival data. Independent PracticeUse the vertical motion formula h = 16t2 + vt + c.A soccer ball is kicked with a starting upward velocity of 50 ft/s from a starting height of 3.5 ft. Substitute the values into the vertical motion formula and let h = 0. Use the quadratic formula to solve for t. If no one touches the ball, how long is the ball in the air?A.0.1 sB.0.1 sC.3.2 sD.1.1 s if you is equals to 1 2 3 4 5 6 7 8 9 10 and a is equal to 1267 b is equals to 2 3 5 6 and C is equals to 4 5 6 7 then verify that a union B complement is equal to a complement intersection b complement A student skipped a step when she tried to convert 18 hours into seconds, and she got the following incorrect result: write at least three difference between arteryand vein with diagram Frankenstein created the monster on a larger scale because the larger components were easier to work with, and he didn't have refined instruments. Select one: True False The gradient of a straight line passes through points (6,0) and (0,q) is -3/2. Find the value of q Geographically India is a________ Which of the following is a cost of using dams as a source of freshwater why are living organisms classified? A blue line with 5 orange tick marks then one red tick mark then 4 orange tick marks. The number zero is above the red tick mark.Assume each tick mark represents 1 cm.Calculate the total displacement from 0 if an object moves 3 cm to the left, then 7 cm to the right, and then 6 cm to the left.The object moves cm to the left.What is the total distance the object travels? cm Describe What You See in the PlacardWhich cause/s of New Imperialism does the Placard represent?Explain Why You Chose the Cause/s? According to the Fundamental Theorem of Algebra, which polynomial function has exactly 8 roots?PLS HELP IM TIMED why e=mc2?why not e=mc3? what is -6^2 equal to? A denser object will usually have a ____ index ofrefraction. Solve 15 = 4(1.6)^x by graphing. Round to nearest hundredth. n Office Manager uses a Periodic Review Inventory System: they check the inventory in their Office Supply Closet once every 10 days, placing an order with their supplier depending on the inventory level of the office supplies. The manager has set a restocking level of 300 post it notes for their closet. This week, the manager has counted 140 post it notes in the closet. How many post it notes will the manager order from their supplier? In other words, what is the Order Quantity?